Show that the quotient group $G/N$ contains a subgroup isomorphic to $H$. [closed]












1












$begingroup$



Let $G$ be a finite group, $Nmathrel{lhd}G$ a normal subgroup of $G$, and $Hleq G$ a subgroup of $G$. Suppose that $|H|$ and $|N|$ are relatively prime (i.e., $gcd(|H|,|N|)=1$). Show that the quotient group $G/N$ contains a subgroup isomorphic to $H$.




I am trying to show that there is an injective homomorphism $varphi:Hrightarrow G/N$ , but I have no clue for the next step.










share|cite|improve this question











$endgroup$



closed as unclear what you're asking by Shaun, user10354138, Cesareo, Rebellos, GNUSupporter 8964民主女神 地下教會 Dec 9 '18 at 14:05


Please clarify your specific problem or add additional details to highlight exactly what you need. As it's currently written, it’s hard to tell exactly what you're asking. See the How to Ask page for help clarifying this question. If this question can be reworded to fit the rules in the help center, please edit the question.














  • 4




    $begingroup$
    Do You mean $N triangleleft G$ instead of $H triangleleft G$? And also $G/N$ instead of $G/H$? as $G/H$ may not be a group.
    $endgroup$
    – nafhgood
    Dec 8 '18 at 22:30












  • $begingroup$
    Hint. Think about what the natural homomorphism from $G$ to $G/N$ does to $H$. Can it send anything other than the identity to the identity?
    $endgroup$
    – Ethan Bolker
    Dec 8 '18 at 23:03






  • 1




    $begingroup$
    The second isomorphism theorem says $HN/N cong H/(H cap N)$. Here $H cap N$ is trivial as $(|H|,|N|)=1$. So then $HN/N cong H$.
    $endgroup$
    – nafhgood
    Dec 8 '18 at 23:16
















1












$begingroup$



Let $G$ be a finite group, $Nmathrel{lhd}G$ a normal subgroup of $G$, and $Hleq G$ a subgroup of $G$. Suppose that $|H|$ and $|N|$ are relatively prime (i.e., $gcd(|H|,|N|)=1$). Show that the quotient group $G/N$ contains a subgroup isomorphic to $H$.




I am trying to show that there is an injective homomorphism $varphi:Hrightarrow G/N$ , but I have no clue for the next step.










share|cite|improve this question











$endgroup$



closed as unclear what you're asking by Shaun, user10354138, Cesareo, Rebellos, GNUSupporter 8964民主女神 地下教會 Dec 9 '18 at 14:05


Please clarify your specific problem or add additional details to highlight exactly what you need. As it's currently written, it’s hard to tell exactly what you're asking. See the How to Ask page for help clarifying this question. If this question can be reworded to fit the rules in the help center, please edit the question.














  • 4




    $begingroup$
    Do You mean $N triangleleft G$ instead of $H triangleleft G$? And also $G/N$ instead of $G/H$? as $G/H$ may not be a group.
    $endgroup$
    – nafhgood
    Dec 8 '18 at 22:30












  • $begingroup$
    Hint. Think about what the natural homomorphism from $G$ to $G/N$ does to $H$. Can it send anything other than the identity to the identity?
    $endgroup$
    – Ethan Bolker
    Dec 8 '18 at 23:03






  • 1




    $begingroup$
    The second isomorphism theorem says $HN/N cong H/(H cap N)$. Here $H cap N$ is trivial as $(|H|,|N|)=1$. So then $HN/N cong H$.
    $endgroup$
    – nafhgood
    Dec 8 '18 at 23:16














1












1








1





$begingroup$



Let $G$ be a finite group, $Nmathrel{lhd}G$ a normal subgroup of $G$, and $Hleq G$ a subgroup of $G$. Suppose that $|H|$ and $|N|$ are relatively prime (i.e., $gcd(|H|,|N|)=1$). Show that the quotient group $G/N$ contains a subgroup isomorphic to $H$.




I am trying to show that there is an injective homomorphism $varphi:Hrightarrow G/N$ , but I have no clue for the next step.










share|cite|improve this question











$endgroup$





Let $G$ be a finite group, $Nmathrel{lhd}G$ a normal subgroup of $G$, and $Hleq G$ a subgroup of $G$. Suppose that $|H|$ and $|N|$ are relatively prime (i.e., $gcd(|H|,|N|)=1$). Show that the quotient group $G/N$ contains a subgroup isomorphic to $H$.




I am trying to show that there is an injective homomorphism $varphi:Hrightarrow G/N$ , but I have no clue for the next step.







group-theory finite-groups normal-subgroups group-isomorphism quotient-group






share|cite|improve this question















share|cite|improve this question













share|cite|improve this question




share|cite|improve this question








edited Dec 8 '18 at 23:13









Batominovski

33k33293




33k33293










asked Dec 8 '18 at 22:29









aaron wangaaron wang

162




162




closed as unclear what you're asking by Shaun, user10354138, Cesareo, Rebellos, GNUSupporter 8964民主女神 地下教會 Dec 9 '18 at 14:05


Please clarify your specific problem or add additional details to highlight exactly what you need. As it's currently written, it’s hard to tell exactly what you're asking. See the How to Ask page for help clarifying this question. If this question can be reworded to fit the rules in the help center, please edit the question.









closed as unclear what you're asking by Shaun, user10354138, Cesareo, Rebellos, GNUSupporter 8964民主女神 地下教會 Dec 9 '18 at 14:05


Please clarify your specific problem or add additional details to highlight exactly what you need. As it's currently written, it’s hard to tell exactly what you're asking. See the How to Ask page for help clarifying this question. If this question can be reworded to fit the rules in the help center, please edit the question.










  • 4




    $begingroup$
    Do You mean $N triangleleft G$ instead of $H triangleleft G$? And also $G/N$ instead of $G/H$? as $G/H$ may not be a group.
    $endgroup$
    – nafhgood
    Dec 8 '18 at 22:30












  • $begingroup$
    Hint. Think about what the natural homomorphism from $G$ to $G/N$ does to $H$. Can it send anything other than the identity to the identity?
    $endgroup$
    – Ethan Bolker
    Dec 8 '18 at 23:03






  • 1




    $begingroup$
    The second isomorphism theorem says $HN/N cong H/(H cap N)$. Here $H cap N$ is trivial as $(|H|,|N|)=1$. So then $HN/N cong H$.
    $endgroup$
    – nafhgood
    Dec 8 '18 at 23:16














  • 4




    $begingroup$
    Do You mean $N triangleleft G$ instead of $H triangleleft G$? And also $G/N$ instead of $G/H$? as $G/H$ may not be a group.
    $endgroup$
    – nafhgood
    Dec 8 '18 at 22:30












  • $begingroup$
    Hint. Think about what the natural homomorphism from $G$ to $G/N$ does to $H$. Can it send anything other than the identity to the identity?
    $endgroup$
    – Ethan Bolker
    Dec 8 '18 at 23:03






  • 1




    $begingroup$
    The second isomorphism theorem says $HN/N cong H/(H cap N)$. Here $H cap N$ is trivial as $(|H|,|N|)=1$. So then $HN/N cong H$.
    $endgroup$
    – nafhgood
    Dec 8 '18 at 23:16








4




4




$begingroup$
Do You mean $N triangleleft G$ instead of $H triangleleft G$? And also $G/N$ instead of $G/H$? as $G/H$ may not be a group.
$endgroup$
– nafhgood
Dec 8 '18 at 22:30






$begingroup$
Do You mean $N triangleleft G$ instead of $H triangleleft G$? And also $G/N$ instead of $G/H$? as $G/H$ may not be a group.
$endgroup$
– nafhgood
Dec 8 '18 at 22:30














$begingroup$
Hint. Think about what the natural homomorphism from $G$ to $G/N$ does to $H$. Can it send anything other than the identity to the identity?
$endgroup$
– Ethan Bolker
Dec 8 '18 at 23:03




$begingroup$
Hint. Think about what the natural homomorphism from $G$ to $G/N$ does to $H$. Can it send anything other than the identity to the identity?
$endgroup$
– Ethan Bolker
Dec 8 '18 at 23:03




1




1




$begingroup$
The second isomorphism theorem says $HN/N cong H/(H cap N)$. Here $H cap N$ is trivial as $(|H|,|N|)=1$. So then $HN/N cong H$.
$endgroup$
– nafhgood
Dec 8 '18 at 23:16




$begingroup$
The second isomorphism theorem says $HN/N cong H/(H cap N)$. Here $H cap N$ is trivial as $(|H|,|N|)=1$. So then $HN/N cong H$.
$endgroup$
– nafhgood
Dec 8 '18 at 23:16










1 Answer
1






active

oldest

votes


















2












$begingroup$

Hint: Show that $HN$ is a subgroup of $G$ containing $N$. Prove that, under the canonical projection $Gto (G/N)$, the subgroup $HN$ of $G$ is mapped onto a subgroup of $G/N$ isomorphic to $H$ (this subgroup is clearly, $HN/N$). More generally, we have $HN/Ncong H/(Hcap N)$.






share|cite|improve this answer









$endgroup$




















    1 Answer
    1






    active

    oldest

    votes








    1 Answer
    1






    active

    oldest

    votes









    active

    oldest

    votes






    active

    oldest

    votes









    2












    $begingroup$

    Hint: Show that $HN$ is a subgroup of $G$ containing $N$. Prove that, under the canonical projection $Gto (G/N)$, the subgroup $HN$ of $G$ is mapped onto a subgroup of $G/N$ isomorphic to $H$ (this subgroup is clearly, $HN/N$). More generally, we have $HN/Ncong H/(Hcap N)$.






    share|cite|improve this answer









    $endgroup$


















      2












      $begingroup$

      Hint: Show that $HN$ is a subgroup of $G$ containing $N$. Prove that, under the canonical projection $Gto (G/N)$, the subgroup $HN$ of $G$ is mapped onto a subgroup of $G/N$ isomorphic to $H$ (this subgroup is clearly, $HN/N$). More generally, we have $HN/Ncong H/(Hcap N)$.






      share|cite|improve this answer









      $endgroup$
















        2












        2








        2





        $begingroup$

        Hint: Show that $HN$ is a subgroup of $G$ containing $N$. Prove that, under the canonical projection $Gto (G/N)$, the subgroup $HN$ of $G$ is mapped onto a subgroup of $G/N$ isomorphic to $H$ (this subgroup is clearly, $HN/N$). More generally, we have $HN/Ncong H/(Hcap N)$.






        share|cite|improve this answer









        $endgroup$



        Hint: Show that $HN$ is a subgroup of $G$ containing $N$. Prove that, under the canonical projection $Gto (G/N)$, the subgroup $HN$ of $G$ is mapped onto a subgroup of $G/N$ isomorphic to $H$ (this subgroup is clearly, $HN/N$). More generally, we have $HN/Ncong H/(Hcap N)$.







        share|cite|improve this answer












        share|cite|improve this answer



        share|cite|improve this answer










        answered Dec 8 '18 at 23:11









        BatominovskiBatominovski

        33k33293




        33k33293















            Popular posts from this blog

            Le Mesnil-Réaume

            Ida-Boy-Ed-Garten

            web3.py web3.isConnected() returns false always